Answer (B) is correct . The contribution margin (CM) for XY-7 is $1?per unit ($4 selling price – $3 unit variable cost). Thus, 160,000 units of XY-7 will generate an additional $160,000 of CM, which is sufficient to cover the increase in advertising costs.
Answer (A) is incorrect because This number of units would be the result if the UCM for XY-7 were $.25 instead of $1.00 (640,000 ¡Á $.25 = $160,000). Fixed manufacturing costs are not included in determining UCM. Answer (C) is incorrect because This number of units implies a $1.25 UCM. Variable selling costs are included and fixed manufacturing costs are not included in determining UCM. Answer (D) is incorrect because This number of units implies a $2.00 UCM. The correct UCM of $1.00 is found by subtracting all variable costs from the selling price.
|